APEA NURSING NR 509

APEA NURSING NR 509

Question:

A transient ischemic attack is:

a transient episode of neurologic dysfunction by focal brain, spinal cord, or retinal ischemia, without acute infarction.   Correctan infarction of the central nervous system tissue that may be silent or symptomatic.the abrupt onset of motor or sensory deficits.  Incorrectfocal or asymmetric weaknesses caused by central and peripheral nerve damage.

ORDER A PLAGIARISM-FREE PAPER HERE

Explanation:

TIA is now defined as “a transient episode of neurological dysfunction caused by focal brain, spinal cord, or retinal ischemia, without acute infarction.” Ischemic stroke is “an infarction of central nervous system tissue” that may be symptomatic or silent. The other terms are not related to the new definitions.

Question:

A patient is asked to visually follow a finger through the cardinal fields of gaze. Which cranial nerves are being assessed?

 

III, IVIII, IV, VI  CorrectV, VI, VII APEA NURSING NR 509

 

Explanation:

Visually following a finger through the cardinal fields of gaze is one way to assess the oculomotor (CN III), trochlear (CN IV) and abducens (CN VI) nerves. CN I is the olfactory nerve and assesses smell. CN II is the optic nerve and assesses visual acuity. CN V is the trigeminal nerve and assesses both sensory and motor functions.

Question:

Uncontrolled electrical activity in the brain, which may produce minor physical signs, thought disturbances, or disturbed motor activity is:

 

dystonia.bradykinesia.tremor.seizure.  Correct

 

Explanation:

A seizure is an uncontrolled electrical activity in the brain which may produce minor physical signs, thought disturbances, or disturbed motor activity, or a combination of symptoms. Dystonia is a neurological disorder that causes involuntary muscle spasms and twisting of the limbs. Bradykinesia is the term used to describe the impaired ability to adjust to one’s body position. This symptom is noted in patients who have Parkinson’s disease. A rhythmic oscillatory movement of a body part resulting from the contraction of opposing muscle groups is a tremor.

Question:

When evaluating the sensory system, testing the posterior columns tract would include assessing sensations of:

 

position and vibration.  Correctpain and temperature.deep touch.  Incorrectdiscriminative sensations.

 

Explanation:

When evaluating the sensory system, testing the spinothalamic tracts would include assessing sensations of pain and temperature. Assessing position and vibration evaluate the posterior columns. Light touch assesses both the spinothalamic and posterior column tracts. To assess discriminative sensation, both the spinothalamic and posterior columns tracts as well as the cortex would be assessed.

Question:

Physical exam of a well two-week-old infant reveals a little dimple with a small amount of hair just above the sacral area. This could be:

 

an unusual finding but within normal limits.  Incorrecthirsutism.Arnold -Chiari malformation. spina bifida occulta.  Correct

 

APEA NURSING NR 509 Explanation:

There are four types of spina bifida: occulta, closed neural tube defects, meningocele, and myelomeningocele. Occulta is the mildest and most common form in which one or more vertebrae are malformed. The name “occulta,” which means “hidden,” indicates that a layer of skin covers the malformation, or opening in the vertebrae. This form of spina bifida, present in 10-20 percent of the general population, rarely causes disability or symptoms. Closed neural tube defects are often recognized early in life due to an abnormal tuft or clump of hair or a small dimple or birthmark on the skin at the site of the spinal malformation. Meningocele and myelomeningocele generally involve a fluid-filled sac—visible on the back—protruding from the spinal canal. In meningocele, the sac may be covered by a thin layer of skin. In most cases of myelomeningocele, there is no layer of skin covering the sac and an area of abnormally developed spinal cord tissue is usually exposed. Hirsutism is an excessive amount of hair on the body usually caused by a hormonal imbalance. An Arnold-Chiari malformation is a cyst-like formation in the fourth ventricle, cerebellum or brainstem.

Question:

A female patient complains of weakness in her arm when combing her hair. This finding could be suggestive of which type of weakness pattern?

 

Proximal  CorrectDistal  IncorrectSymmetricAsymmetric

 

Explanation:

To identify proximal weakness, ask about difficulty with movements such as combing hair, reaching up to a shelf, getting up out of a chair, or climbing a high step.

Question:

What geriatric condition is characterized by normal alertness but progressive global deterioration of cognition in multiple domains?

 

DeliriumCognitive impairmentParkinson’s diseaseAlzheimer’s disease  Correct APEA NURSING NR 509

 

Explanation:

Alzheimer’s disease is a geriatric condition in which normal alertness is present but progressive global deterioration of cognition occurs in multiple domains, including short-term memory, but with sparing of memory for remote events, subtle language errors, visuospatial perceptual difficulties, and changes in executive function, or the ability to perform sequential tasks such as instrumental activities of daily living (IADLs). Delirium is a serious disturbance in a person’s mental abilities that results in a decreased awareness of one’s environment and confused thinking. Cognitive impairment is an intermediate stage between the expected cognitive decline of normal aging and the more serious decline of dementia. Parkinson’s disease is a progressive disorder of the nervous system that affects movement.

Question:

When assessing coordination of muscle movement, four areas of the nervous system function in an integrated way. These areas include the motor, cerebellar, the vestibular, and the sensory systems. Which system coordinates position sense?

 

Motor systemCerebellar systemVestibular system  IncorrectSensory system  Correct

 

Explanation:

Coordination of muscle movement requires that four areas of the nervous system function in an integrated way: motor system for muscle strength, cerebellar system for rhythmic movements and steady posture, vestibular system for balance and coordinating eye, head, and body movements, and sensory system for position sense.

Question:

A band of skin innervated by the sensory root of a single spinal nerve is termed a:

 

peripheral nerve field.dermatome.  Correctsynapseasterixis.

 

Explanation:

A band of skin innervated by the sensory root of a single spinal or dorsal nerve root is termed a dermatome. A peripheral nerve field refers to an area of the skin innervated by a single nerve and is described as cutaneous nerve distribution. A synapse is a structure that permits a neuron to pass an electrical or chemical signal to another cell. Asterixis refers to an abnormal tremor consisting of involuntary jerking movements, especially in the hands, frequently occurring with impending hepatic coma and other forms of metabolic encephalopathy. This is also known as flapping tremor. APEA NURSING NR 509

Question:

An 8-month-old with a significant head lag would suggest the need for:

 

exercises that strengthen the neck muscles.a follow-up visit in 2 months.a neurological evaluation.  Correctan orthopedic referral.

 

Explanation:

A baby typically should be able to control the neck muscles by around 4 months of age, so that when pulling from a lying to a sitting position the head should remain in line with the torso and not flop back. Delays in reaching this developmental milestone have been observed in premature babies and in those with cerebral palsy; all others should be referred to a pediatric neurologist for further evaluation. The other choices do not address the issue or the cause. Early interventions result in better outcomes.

Question:

Hyperalgesia refers to:

 

decreased sensitivity to pain.increased sensitivity to pain.  Correctabsence of pain sensation.absence of touch sensation.

 

Explanation:

Analgesia refers to absence of pain sensation; hypalgesia to decreased sensitivity to pain; hyperalgesia to increased sensitivity; and anesthesia to absence of touch sensation.

Question:

Brief, repetitive, stereotyped, coordinated movements occurring at irregular intervals are consistent with:

 

facial tics.  Correctdystonic movements.athetoid movements.oral-facial dyskinesias.  Incorrect

 

Explanation:

Facial tics are brief, repetitive, stereotyped, coordinated movements occurring at irregular intervals. Examples include repetitive winking, grimacing, and shoulder shrugging. Dystonic movements are similar to athetoid movements, but often involve larger portions of the body, including the trunk. Grotesque, twisted postures may result. Athetoid movements are slower and more twisting and writhing than choreiform movements, and have a larger amplitude. They most commonly involve the face and the distal extremities. Oral–facial dyskinesias are rhythmic, repetitive, bizarre movements that chiefly involve the face, mouth, jaw, and tongue: grimacing, pursing of the lips, protrusions of the tongue, opening and closing of the mouth, and deviations of the jaw. These are involuntary movements.

Question:

When a patient complains of severe headaches that have worsened over the last few weeks but she has no other symptoms, a most likely diagnosis would be:

 

sinusitis.a tumor.  Correctsubarachnoid hemorrhage.  Incorrectan abscess.

 

Explanation:

The most important attributes of headaches are chronologic pattern and severity. Changing or progressively severe headaches increase the likelihood of tumor, abscess, or other masses. Extremely severe headaches suggest subarachnoid hemorrhage or meningitis and these headaches require immediate intervention because they worsen rapidly, not over a “few weeks”. Sinusitis does cause headache but these headaches are not usually severe.

Question:

While assessing the cranial nerves, the nurse practitioner touches the cornea lightly with a wisp of cotton. This maneuver assesses which cranial nerve?

 

Cranial Nerve II (CN II)Cranial Nerve IV (CN IV)  IncorrectCranial Nerve V (CN V)  CorrectCranial Nerve X (CN X)

 

Explanation:

Touching the cornea and assessing for a reflex assesses cranial nerve V.

Question:

The patient has his eyes closed and an area on his right leg is briefly touched by the nurse practitioner. The patient is instructed to open his eyes and point to the area that was touched. This is an example of the discriminative sensation known as:

 

graphesthesia.stereognosis.two point discrimination.  Incorrectpoint localization.  Correct

 

Explanation:

Graphesthesia, or number identification is the ability to identify a number when drawn in the hand of a patient whose eyes are closed. A discriminative sensation, stereognosis, is the ability to identify an object by feeling. The ability to identify an object touching 2 areas simultaneously is termed two-point discrimination. Point localization refers to the ability of the patient to identify the area of the body that was touched when his eyes were closed.

Question:

When assessing the cranial nerves, the nurse practitioner observes that the soft palate does not rise when the patient is instructed to say “ah”. This finding could be suggestive of a bilateral lesion in which cranial nerve?

 

Cranial Nerve V (CN V)Cranial Nerve VII (CN VII)Cranial Nerve IX (CN IX)Cranial Nerve X (CN X)  Correct

 

Explanation:

Failure of the soft palate to rise when the patient is instructed to say “ah” or yawn could be suggestive of a bilateral lesion in cranial nerve X (CN X)-vagus nerve.

Question:

The level of consciousness that refers to the patient that remains unarousable with eyes closed without evidence of response to inner need or external stimuli is said to be in:

 

an obtunded state.a comatose state.  Correcta lethargic state.a stuporous state.

 

Explanation:

A patient that remains unarousable with eyes closed without evidence of response to inner need or external stimuli is a comatose patient. An obtunded patient opens his eyes, looks at the person speaking to him but responds slowly and appears confused. Lethargy refers to the patient that appears drowsy but can open his eyes, respond to questions, then fall back to sleep. A stuporous patient arouses from sleep only after painful stimuli.

Question:

Having the patient shrug his shoulders and elicit neck movements would be testing Cranial Nerve:

 

VI.VII.IX.XI.  Correct

 

Explanation:

Cranial Nerve XI is the spinal accessory nerve and is responsible for proper functioning of the shoulder and neck muscles. When the trapezius is paralyzed, the shoulder droops and the scapula is displaced downward and laterally. Weakness with atrophy and fasciculations indicates a peripheral nerve disorder. CN VI tests extraocular movements; CN VII tests hearing; and CN IX and X test swallowing and the gag reflex. APEA NURSING NR 509

Question:

The Glasgow coma scale assesses:

 

cranial nerve response.pupillary response.motor response.  Correctauditory response.

 

Explanation:

Based on motor responsiveness, verbal performance, and eye opening to appropriate stimuli, the Glasgow Coma Scale was designed and should be used to assess the depth and duration of coma and impaired consciousness. This scale helps to gauge the impact of a wide variety of conditions such as acute brain injury due to traumatic and/or vascular injuries or infections, metabolic disorders such as hepatic or renal failure, hypoglycemia, or diabetic ketoacidosis. Cranial nerve, auditory, and pupillary responses are not included in the come scale assessment.

Question:

Discriminative sensations include all of the following except:

 

astereognosis.stereognosis.graphesthesia.deep tendon reflexes  Correct

 

Explanation:

Discriminative sensations test the ability of the sensory cortex to correlate, analyze, and interpret sensations. These include: stereognosis, graphesthesia, two-point identification, point localization, and extinction. Deep tendon reflexes evaluate the spinal nerve roots and usually include C5, C6, C7, L4, and S1.

Question:

Symmetric weakness of the distal muscles of the legs suggests a:

 

polyneuropathy.  Correctmyopathy.sensory neuropathy.cerebellar disease.  Incorrect

 

Explanation:

Polyneuropathy would present as symmetric weakness in the distal muscles. Symmetric weakness of the proximal muscles of the legs suggests a myopathy. Nystagmus, dysarthria, hypotonia, and ataxia would be characteristic of cerebellar disease. Sensory neuropathy usually presents as numbness, tingling, and pain.

Question:

The thalamus and the basal ganglion are located in the:

 

spinal cord.peripheral nervous system.white matter.gray matter.  Correct

 

Explanation:

Deep in the brain lie additional clusters of gray matter. These include the basal ganglia, which affect movement, and the thalamus and the hypothalamus structures in the diencephalon. The thalamus processes sensory impulses and relays them to the cerebral cortex. APEA NURSING NR 509.

Question:

The most common cause of viral encephalitis in children is:

 

Herpes simplex virus Type IIPicornavirusEnteroviruses  IncorrectHerpes simplex virus Type I  Correct

 

Explanation:

Herpes simplex Type I is the most common cause of viral encephalitis in children. The other choices are incorrect.

Question:

By placing the patient in the supine position, the nurse practitioner raises the patient’s relaxed and straightened leg while flexing the leg at the hip, then dorsiflexes the foot. This maneuver is known as:

 

Kernig’s sign.the straight-leg raise.  Correctthe plantar response.the ankle reflex.  Incorrect

 

Explanation:

By placing the patient in the supine position, the nurse practitioner raises the patient’s relaxed and straightened leg while flexing the leg at the hip, then dorsiflexes the foot. This maneuver is known as the straight leg raise and is used to evaluate sciatica. It is positive if there is pain down the back of the leg below the knee. Ipsilateral calf wasting and weak ankle dorsiflexion may be present.

Question:

With the adult patient lying supine, the nurse practitioner flexes the patient’s neck while observing the hips and knees. Flexion of both hips and knees was noted. This is a positive:

 

Brudzinski’s sign.  CorrectKernig’s sign.nuchal rigidity sign.Babinski’s sign.

 

Explanation:

To elicit Brudzinski’s sign, flex the neck. Flexion of both the hips and knees is positive for Brudzinski’s sign. To test for Kernig’s sign, flex the patient’s leg at both the hip and the knee and then straighten the knee. Pain and increased resistance to extending the knee are positive for Kernig’s sign. With the patient lying supine, the nurse practitioner places her hands behind the patient’s head while flexing his neck forward so that his chin touches his chest. Neck stiffness with resistance to flexion is noted. This is positive for nuchal rigidity and suggestive of meningeal inflammation from meningitis or subarachnoid hemorrhage. To elicit the Babinski response, stroke the lateral aspect of the sole from the heel to the ball of the foot with the end of an applicator stick; plantar flexion is normal. Dorsiflexion of the big toe is a positive Babinski’s sign.

Question:

When a two-week-old infant presents with irritability, poor appetite, and rapid head growth with distended scalp veins, one should consider:

 

hydrocephalus.  Correctmeningitis.cerebral palsy.Reye’s syndrome. APEA NURSING NR 509.

 

Explanation:

The combination of signs is strongly suggestive of hydrocephalus: shrill with high-pitched cry, irritability, tense and bulging fontanels due to the increased amount of CSF being produced or not being absorbed. Meningitis would include signs of sepsis/infection. The manifestations of cerebral palsy vary but may include: persistence of primitive reflexes, delayed gross motor development, and a lack of progression through developmental milestones. Reye’s syndrome is associated with an antecedent viral infection with symptoms of malaise, nausea, and vomiting, progressive neurological deterioration occurs.

Question:

A term used to describe muscle wasting or loss of muscle bulk is:

 

hypertrophy.muscular atrophy.  Correctpseudohypertrophy.muscle weakness.

 

Explanation:

A term used to describe muscle wasting or loss of muscle bulk is muscular atrophy. Hypertrophy refers to an increase in bulk of the muscle with proportionate strength. An increase in muscular bulk with diminished strength is known as pseudohypertrophy. Muscular weakness is a term used to describe a lack of strength or firmness in a muscle.

Question:

Which one of the following senses is most often affected in patients on long-term antibiotic therapy?

 

TouchTasteSmellHearing  Correct

 

Explanation:

Many antibiotics can produce varying degrees of ototoxicity. The other senses are rarely, if ever, affected by long – term antibiotic use.

Question:

Common physical findings in a young child with cerebral palsy include which one of the following?

 

Walks by placing the heels of the feet down firstMoves about by crawling on the abdomen or all four extremitiesGenerally meets motor developmental milestones on schedulePresence of crossed or touching knees  Correct

 

Explanation:

Cerebral palsy (CP) is a group of disorders that can involve the brain and nervous system functions, such as movement, learning, hearing, seeing, and thinking. There are several different types of cerebral palsy, including spastic, dyskinetic, ataxic, hypotonic, and mixed. Symptoms usually depend on the type and can be seen before a child is 2 years old, and sometimes as early as 3 months. Symptoms may include delays in reaching and in developmental stages such as sitting, rolling, crawling, or walking, or abnormal gait. Arms may be tucked in toward the sides, knees may be crossed or touching, legs may make “scissor” movements, and child may walk on toes. Additionally, newborn reflexes may persist beyond the expected time frame for their disappearance.

Question:

The principal muscles involved when closing the mouth are innervated by which Cranial nerve?

 

Cranial nerve III (CN III)Cranial nerve V (CN V)  CorrectCranial nerve VII (CN VII)  IncorrectCranial nerve XII (CN XII)

 

Explanation:

The muscles responsible for closing the mouth are innervated by Cranial nerve V (trigeminal nerve). Cranial nerve V innervates the masseter, the temporalis, and the internal pterygoids.

Question:

The term used to describe involuntary muscle spasms and twisting of the limbs is:

 

dystonia.  Correctbradykinesia.akinesia.  Incorrectdyskinesia.

 

Explanation:

Dystonia is a neurological disorder that causes involuntary muscle spasms and twisting of the limbs. Bradykinesia is the term used to describe the impaired ability to adjust to one’s body position. This symptom is noted in patients who have Parkinson’s disease. The absence or loss of control of voluntary muscle movements is akinesia. Dyskinesia is defined as the presence of involuntary muscle movements such as tics or chorea. These movements can be seen in children who have rheumatic fever.

Question:

Postural tremors appear when the affected part is:

 

at rest.moving voluntarily. is actively maintaining a posture.  Correctgetting closer to its target.

 

Explanation:

Tremors are rhythmic oscillatory movements. Postural tremors appear when the affected part is actively maintaining a posture. Examples include the fine rapid tremor of hyperthyroidism, the tremors of anxiety and fatigue, and benign essential tremor. The other choices are not consistent with postural tremors.

Question:

A patient presents with sweating, tremors, palpitations, hunger, and confusion. This patient is most likely experiencing:

 

a syncopic event.hypoglycemia.  Correctpostural hypotension.hypocapnia.

 

Explanation:

A patient presenting with sweating, tremors, palpitations, hunger, headache, abnormal behavior, confusion, could be experiencing classic symptoms of hypoglycemia. Syncope could present with nausea and vomiting, dizziness, and possible fainting. Postural hypotension usually occurs after one stands up. Usually there are no prodromal symptoms. The blood pressure returns to normal when the patient lies down. Hypocapnia is decreased carbon dioxide and symptoms include dyspnea, palpitations, chest discomfort, numbness and tingling in the hands and around the mouth lasting for several minutes. Consciousness is maintained.

Question:

Most peripheral nerves contain afferent and efferent fibers. The term afferent refers to:

 

the cranial nerve fibers.spinal nerve fibers.sensory nerve fibers.  Correctmotor nerve fibers.

 

Explanation:

The peripheral nervous system includes spinal and peripheral nerves that carry impulses to and from the cord. Spinal nerve fibers co-mingle with similar fibers from other levels in plexuses outside the cord, from which peripheral nerves emerge. Most peripheral nerves contain both sensory (afferent) and motor (efferent) fibers.

Question:

During this type of seizure activity, the patient loses consciousness suddenly, sometimes with a cry, and the body stiffens into tonic extensor rigidity. Breathing stops, and the person becomes cyanotic. A clonic phase of rhythmic muscular contraction follows. This type of seizure activity is referred to as a:

 

focal seizure with impairment of consciousness.Jacksonian seizure.focal seizure that become generalized. grand mal seizure.  Correct

 

Explanation:

During a grand mal seizure the person loses consciousness suddenly, sometimes with a cry, and the body stiffens into tonic extensor rigidity. Breathing stops, and the person becomes cyanotic. A clonic phase of rhythmic muscular contraction follows. Focal seizures that become generalized are partial seizures that resemble tonic-clonic seizures. The patient may recall the aura and a unilateral neurologic deficit is present during the postictal period. Focal seizures with impairment of consciousness the person appears confused. Automatisms include automatic motor behaviors such as chewing, smacking the lips, walking about, and unbuttoning clothes. Tonic and then clonic movements that start unilaterally in the hand, foot, or face and spread to other body parts on the same side with the patient remaining conscious are known as Jacksonian seizures.

Question:

When assessing plantar reflexes, the nurse practitioner strokes the lateral aspect of the sole from the heel to the ball of the right foot. Absence of movement of the big toe is noted. This finding could be suggestive of a pathologic lesion in which segmented level of the spine?

 

Thoracic 8, 9, and 10Thoracic 10, 11, and 12Lumbar 5 and Sacral 1  CorrectSacral 2, 3, and 4

 

Explanation:

Superficial (cutaneous) reflexes and their corresponding spinal segments include the following: Abdominal reflexes: upper thoracic 8, 9, 10 and lower thoracic 10, 11, 12; Plantar: lumbar 5 and sacral 1; and Anal: sacral 2, 3, 4.

Question:

A patient is instructed to stand, close both eyes, and extend both arms forward with the palms facing upward for 20-30 seconds. If the forearm drifts downward, this would indicate:

 

a normal finding.lower motor neuron disease.corticospinal tract lesion .  Correctcerebellum lesion.

 

Explanation:

A patient is instructed to stand, close both eyes, and extend both arms forward with the palms facing upward for 20-30 seconds. If the forearm drifts downward, this would indicate a corticospinal lesion originating in the contralateral hemisphere. An upward drift would be indicative of a lesion in the cerebellum. This test is called the Test for Pronator Drift and is used to test for upper motor neuron disease. APEA NURSING NR 509.

Question:

When evaluating a patient for weakness of the upper extremities, bilateral distal weakness is noted. This finding could be suggestive of:

 

alcohol myopathy.polyneuropathy.  Correctmyositis.neuromuscular junction disorders.  Incorrect

 

Explanation:

Bilateral predominantly distal weakness suggests a polyneuropathy, as in diabetes. Proximal limb weakness, usually symmetric and without sensory loss, occurs in myopathies from alcohol, glucocorticoids, and inflammatory muscle disorders like myositis and dermatomyositis. In the neuromuscular junction disorder myasthenia gravis, there is proximal, typically asymmetric weakness that gets worse with effort.

Question:

Ptosis of the left eye would be suggestive of damage to which cranial nerve?

 

Cranial Nerve II (CN II)Cranial Nerve III (CN III)  CorrectCranial Nerve IV (CN IV)Cranial Nerve V (CN V)

 

Explanation:

Ptosis of the left eye would be suggestive of 3rd nerve palsy (CN III)-Oculomotor nerve.

Question:

While palpating the temporal and masseter muscles, the patient is asked to clench his teeth and move his jaw from side to side. This maneuver would be assessing which cranial nerve?

 

Cranial Nerve III (CN III)Cranial Nerve IV (CN IV)Cranial Nerve V (CN V)  CorrectCranial Nerve VII (CN VII)  Incorrect

 

Explanation:

Palpation of the temporal and masseter muscles, when the patient clenches his teeth and moves his jaw from side to side. This maneuver assesses the Trigeminal nerve and cranial nerve (CN V).

Question:

With the adult patient lying supine, the nurse practitioner strokes the lateral aspect of the sole from the heel to the ball of the foot with the end of an applicator stick. Dorsiflexion of the big toe was noted. This is a positive:

 

Brudzinski’s sign.Kernig’s sign.nuchal rigidity sign.Babinski’s sign.  Correct

 

Explanation:

To elicit Babinski sign, stroke the lateral aspect of the sole from the heel to the ball of the foot with the end of an applicator stick. Plantar flexion is normal. Dorsiflexion of the big toe is a positive Babinski’s sign. To elicit Brudzinski’s sign, flex the neck. Flexion of both the hips and knees is positive for Brudzinski’s. To test for Kernig’s sign, flex the patient’s leg at both the hip and the knee and then straighten the knee. Pain and increased resistance to extending the knee are positive for a Kernig’s sign. With the patient lying supine, the nurse practitioner places her hands behind the patient’s head while flexing his neck forward until his chin touches his chest. Neck stiffness with resistance to flexion is noted. This is positive nuchal rigidity and suggestive of meningeal inflammation from meningitis or subarachnoid hemorrhage.

Question:

An ambulatory child with spastic cerebral palsy needs a diet:

 

high in potassium.low in fiber.low in sodium.high in calories.  Correct

 

Explanation:

Every child’s nutrient needs must be assessed individually. It is important to take a number of factors into consideration when estimating energy needs: age, activity level, growth rate, and muscle tone. A child with spastic cerebral palsy (CP) who is not ambulatory will likely have lower energy requirements than a child with spastic CP who is ambulatory. A child with ambulatory spastic CP needs a diet high in calories to provide for increased metabolic needs secondary to energy lost through spastic movements and tremors. Foods rich in fiber are needed to prevent constipation and other gastrointestinal problems associated with cerebral palsy. Diet with balanced electrolytes is preferred.

Question:

The part of the brain that relays sensory information between brain regions and controls many autonomic functions of the peripheral nervous system is known as the:

 

cerebrum.brainstem.  Incorrectcerebellum.diencephalon.  Correct

 

Explanation:

The diencephalon relays sensory information between brain regions and controls many autonomic functions of the peripheral nervous system. It also connects structures of the endocrine system with the nervous system and works in conjunction with limbic system structures to generate and manage emotions and memories. The cerebrum controls all voluntary actions of the body with the aid of the cerebellum. The nerve connections of the motor and sensory systems from the main part of the brain to the rest of the body pass through the brainstem. The brainstem controls most functions in the body but is mostly responsible for breathing, heart rate, and articulate speech. The cerebellum, which lies at the base of the brain, coordinates all movement and helps maintain the body upright in space. APEA NURSING NR 509.

Question:

Dysarthria refers to:

 

the inability to produce or understand language.the loss of voice.an impairment in volume of the voice.a defect in the muscular control of the speech apparatus.  Correct

 

Explanation:

Dysarthria refers to a defect in the muscular control of the speech apparatus (lips, tongue, palate, or pharynx). Dysphonia refers to less severe impairment in the volume, quality, or pitch of the voice. Aphonia refers to a loss of voice that accompanies disease affecting the larynx or its nerve supply. Aphasia refers to a disorder in producing or understanding language.

Question:

The inability to hop in place on each foot, could be suggestive of a:

 

cerebellar dysfunction.  Correctcorticospinal tract weakness.  Incorrectproximal tract weakness.weakness of the quadriceps.

 

Explanation:

Inability to hop in place on each foot could denote cerebellar dysfunction, weakness, or lack of position sense. Hopping involves the proximal muscles of the legs as well as the distal ones and requires both good position sense and normal cerebellar function. Walking on the toes and heels may reveal distal muscular weakness in the legs. Inability to heel-walk is a sensitive test for corticospinal tract weakness. Difficulty performing a shallow knee bend would suggest proximal weakness in the extensors of the hip, weakness of the quadriceps, extensors of the knee, or both.

Question:

Hyperesthesia refers to:

 

absence of touch sensation.decreased sensitivity to touch.increased sensitivity to touch.   Correctabsence of pain sensation.

 

Explanation:

Anesthesia is absence of touch sensation; hypesthesia is decreased sensitivity to touch; hyperesthesia is increased sensitivity to touch; and analgesia refers to absence of pain sensation.

Question:

The part of the brain that maintains homeostasis is the:

 

basal ganglion.thalamus.hypothalamus.  Correctcerebellum.  Incorrect

 

Explanation:

The hypothalamus maintains homeostasis and regulates temperature, heart rate, and blood pressure. The hypothalamus affects the endocrine system and governs emotional behaviors such as anger and sexual drive. Hormones secreted in the hypothalamus act directly on the pituitary gland.

Question:

The patient experiences a sudden loss of consciousness with falling without movements and injury may occur. This type of a seizure is consistent with:

 

a myoclonic seizure.an absent seizure.a myoclonic atonic seizure.  Correcta focal seizure with impairment of consciousness.  Incorrect

 

Explanation:

During a myoclonic atonic seizure, the patient experiences a sudden loss of consciousness with falling but no movements. Injury may occur. A patient experiencing a myoclonic seizure manifests sudden, brief, rapid jerks, involving the trunk or limbs. A sudden brief lapse of consciousness, with momentary blinking, staring, or movements of the lips and hands but no falling is consistent with an absent seizure. Focal seizures with impairment of consciousness the person appears confused. Automatisms include automatic motor behaviors such as chewing, smacking the lips, walking about, and unbuttoning clothes.

Question:

Involuntary movements of the body that are slower and more twisting and writhing than choreiform movements, and have a larger amplitude are suggestive of:

 

facial tics.dystonic movements.athetoid movements.  Correctoral-facial dyskinesias.

 

Explanation:

Athetoid movements are slower and more twisting and writhing than choreiform movements, and have a larger amplitude. They most commonly involve the face and the distal extremities. Facial tics are brief, repetitive, stereotyped, coordinated movements occurring at irregular intervals. Examples include repetitive winking, grimacing, and shoulder shrugging. Dystonic movements are similar to athetoid movements, but often involve larger portions of the body, including the trunk. Grotesque, twisted postures may result. Oral–facial dyskinesias are rhythmic, repetitive, bizarre movements that chiefly involve the face, mouth, jaw, and tongue: grimacing, pursing of the lips, protrusions of the tongue, opening and closing of the mouth, and deviations of the jaw. These are involuntary movements.

Question:

Which of the following procedures should NOT be performed in a comatose patient?

 

Check corneal responseCheck pupillary responseDilate the pupils  CorrectInspect the posterior pharynx  Incorrect

 

Explanation:

When assessing a comatose patient, the nurse practitioner should not dilate the eyes because pupillary reaction is the single most important clue to the underlying cause of the coma: structural or metabolic. The other procedures can be performed on a comatose patient.

Question:

A form of aphasia in which the person has word-finding difficulties for speaking and writing is known as:

 

Broca’s aphasia.  Incorrectanomic aphasia.  CorrectWernicke’s aphasia.global aphasia.

 

Explanation:

With anomic aphasia, the person has word-finding difficulties and because of the difficulties, the person struggles to find the right words for speaking and writing. In Broca’s aphasia, speech is confluent, slow, with few words and laborious effort. Inflection and articulation are impaired but words are meaningful, with nouns, transitive verbs, and important adjectives. Small grammatical words are often dropped. With Wernicke’s aphasia, speech is fluent, often rapid, voluble, and effortless. Inflection and articulation are good, but sentences lack meaning and words are malformed (paraphasias) or invented (neologisms). Speech may be totally incomprehensible. With global aphasia, the person has difficulty speaking and understanding words and is unable to read or write.

Question:

A neurological assessment to evaluate neurologic input to the cerebellum is:

 

Glasgow coma scale.  Incorrectabdominal reflex.babinski test.romberg test.  Correct

 

Explanation:

Romberg test is performed by asking the patient to stand and with his eyes closed and feet together for at least 20 seconds and he should not sway. This test evaluates sensory input to the cerebellum to maintain truncal stability. The sensory inputs: proprioception, vision, and vestibular sense. Glasgow coma scale determines level of consciousness. An absent abdominal reflex could indicate a disease of the upper and lower neurons. A positive Babinski would indicate upper motor neuron disease in the pyramidal tract.

Question:

The three important questions that govern the neurological examination include: Is the mental status intact?, Are right-sided and left-sided findings symmetric?, and:

 

Are deep tendon reflexes intact?.Are vital signs stable?If findings are abnormal, is the cause in the central or peripheral nervous system?.  CorrectAre the cranial nerves intact?.  Incorrect

 

Explanation:

The three important questions that govern the neurological examination include: Is the mental status intact?, Are right-sided and left-sided findings symmetric?, and If findings are abnormal, is the cause in the central or peripheral nervous system?. The other questions are also important to ask but are not included in the 3 most important ones for the neurological exam.

Question:

Which developmental area is predominantly affected by lead poisoning?

 

NutritionCommunicationCognition  CorrectMobility

 

Explanation:

Lead is a naturally-occurring element that can be harmful to humans when ingested or inhaled, particularly to children under the age of 6 years. Lead poisoning can cause a number of adverse human health effects, but is particularly detrimental to the neurological development of children.

Question:

A term used to describe an increase in muscular bulk with diminished strength is:

 

hypertrophy.muscular atrophy.pseudohypertrophy.  Correctmuscle weakness.

 

Explanation:

An increase in muscular bulk with diminished strength is known as pseudohypertrophy. Hypertrophy refers to an increase in bulk of the muscle with a proportionate increase in strength. A term used to describe muscle wasting or loss of muscle bulk is muscular atrophy. Muscular weakness is a term used to describe a lack of strength or firmness in a muscle.

Question:

On examination of the adult patient, symptoms of flexed posture, tremor, rigidity, and shuffling gait are observed. These findings are consistent with:

 

drug induced reaction.functional impairment.Parkinson’s disease.  Correctdepression.

 

Explanation:

Evidence of flexed posture, tremor, rigidity, bradykinesia, micrographia, shuffling gait, and difficulty rising from a chair are symptoms consistent with Parkinson’s disease.

Question:

A form of aphasia where articulation is good but sentences lack meaning is referred to as:

 

Broca’s aphasia.  Incorrectanomic aphasia.Wernicke’s aphasia.  Correctglobal aphasia.

 

Explanation:

With Wernicke’s aphasia, speech is fluent, often rapid, voluble, and effortless. Inflection and articulation are good, but sentences lack meaning and words are malformed (paraphasias) or invented (neologisms). Speech may be totally incomprehensible. In Broca’s aphasia, speech is confluent, slow, with few words and laborious effort. Inflection and articulation are impaired but words are meaningful, with nouns, transitive verbs, and important adjectives. Small grammatical words are often dropped. With anomic aphasia, the person has word-finding difficulties and because of the difficulties, the person struggles to find the right words for speaking and writing. With global aphasia, the person has difficulty speaking and understanding words and is unable to read or write.

Question:

When comparing two sides of the body for symmetric sensation a symmetrical distal sensory loss would be suggestive of :

 

a lesion in the opposite cerebral hemisphere.polyneuropathy.  Correcta spinal cord lesion.  Incorrecta peripheral lesion.

 

Explanation:

When comparing two sides of the body for symmetric sensation a symmetrical distal sensory loss would be suggestive of polyneuropathy. A hemisensory loss pattern would suggest a lesion in the opposite cerebral hemisphere. Spinal cord lesions would present with sensory loss from tract damage below the level of the lesion. A peripheral lesion with sensory loss would present with a stocking – glove distribution.

Question:

When eliciting deep tendon reflexes in the knee, the nurse practitioner notes an abnormal reflex in the right knee. This abnormality is probably consistent with a pathological lesion in which segmented level of the spine?

 

Cervical 5 and 6Cervical 6 and 7 Lumbar 2, 3, and 4  CorrectSacral 1

 

Explanation:

The segmented levels of the deep tendon reflexes are: Ankle: sacral 1; knee: lumbar 2,3, & 4; Supinator and biceps: cervical 5 & 6; and triceps: cervical 6 & 7. APEA NURSING NR 509.

Question:

Persistent blinking after glabellar tap and difficulty walking heel-to- toe are common in:

 

Alzheimer’s disease.Muscular Dystrophy.Parkinson’s disease.  CorrectMultiple Sclerosis.

 

Explanation:

Glabellar tap is a primitive reflex that is characterized by blinking eyes when a patient is lightly tapped between the eyebrows. In less that 5 taps, a normal individual will stop blinking. In Parkinson’s disease, persistent blinking will occur until the examiner stops tapping. Difficulty walking heel-to-toe are common in Parkinson’s disease.

Question:

To evaluate a patient’s response to light touch sensation, the nurse practitioner would ask the patient to identify:

 

a touch on the skin in response to touching the skin with a cotton wisp.  Correctan object as being hot or cold.a vibration sensation on the big toe.pain as sharp or dull when the thumb is touched using the sharp end of a safety pin.

 

Explanation:

A light touch on the skin in response to touching the skin with a cotton wisp would be an example of assessing for light touch. To evaluate for pain, a safety pin could be used to determine if the sensation is sharp or dull. To evaluate a patient’s response to temperature sensation, the nurse practitioner would ask the patient to identify an object as being hot or cold. To test for vibration, use a tuning fork over the interphalangeal joint of the big toe and ask the patient if he feels the vibration.

Question:

A discriminative sensation that describes the ability to identify a number drawn in the hand when the patient’s eyes are closed is:

 

graphesthesia.  Correctstereognosis.two point discrimination.astereognosis.

 

Explanation:

Graphesthesia or number identification is the ability to identify a number when drawn in the hand of a patient whose eyes are closed. A discriminative sensation. stereognosis, is the ability to identify an object by feeling it. The ability to identify an object touching 2 areas simultaneously is termed two-point discrimination. Astereognosis is a term used to describe the inability to recognize objects placed in the hand.

Question:

When assessing the patient’s sense of position, instruct the patient to first stand with his feet together and eyes open, then instruct him to close both eyes for 30-60 seconds. If he loses his balance with his eyes closed, this is:

 

considered a normal finding.suggestive of ataxia related to dorsal column disease.   Correctsuggestive of cerebellar ataxia. corticospinal track damage.  Incorrect

 

Explanation:

When assessing the patient’s sense of position, instruct the patient to first stand with his feet together and eyes open, then instruct him to close both eyes for 30-60 seconds. If he loses his balance with his eyes closed, this is a positive Romberg test and suggestive of ataxia related to a dorsal column disease. In cerebellar ataxia, the patient has difficulty standing with feet together whether the eyes are open or closed. With corticospinal tract damage, the gait is affected and the patient is unable to heel-walk.

Question:

A female patient complaints of weakness in both arms when transferring the wet clothes from the washer and placing them in the dryer. This finding could be suggestive of which type of weakness pattern?

 

ProximalDistalSymmetric  CorrectAsymmetric

 

Explanation:

To identify symmetric weakness, ask about experiencing weakness in the same area on both sides of the body.

Question:

With the patient lying supine, the nurse practitioner places her hands behind the patient’s head while flexing his neck forward until his chin touches his chest. Neck stiffness with resistance to flexion is noted. This is a positive:

 

Brudzinski’s sign.Kernig’s sign.nuchal rigidity sign.  CorrectBabinski’s sign.

 

Explanation:

With the patient lying supine, the nurse practitioner places her hands behind the patient’s head while flexing his neck forward until his chin touches his chest. Neck stiffness with resistance to flexion is positive for nuchal rigidity and suggestive of meningeal inflammation from meningitis or subarachnoid hemorrhage. To elicit Brudzinski’s sign, flex the neck. Flexion of both the hips and knees is a positive for Brudzinski’s sign. To test for Kernig’s sign, flex the patient’s leg at both the hip and the knee and then straighten the knee. Pain and increased resistance to extending the knee are positive for Kernig’s sign. To elicit the Babinski response, stroke the lateral aspect of the sole from the heel to the ball of the foot with the end of an applicator stick, plantar flexion is normal. Dorsiflexion of the big toe is a positive Babinski’s sign.

Question:

When trying to determine the level of consciousness in a patient whose level of consciousness is altered, a lethargic patient:

 

opens the eyes and looks at the examiner, responds slowly, and is somewhat confused.appears drowsy but opens the eyes, looks at the examiners, answers the questions, and then falls asleep.  Correctarouses from sleep after exposure to painful stimuli, exhibits slow verbal responses, and easily lapses into an unresponsive state.  Incorrectremains unarousable with eyes closed.

 

Explanation:

A lethargic patient appears drowsy but opens his eyes, looks at the examiners, answers the questions, and then falls asleep. An obtunded patient opens the eyes and looks at the examiner, but responds slowly and is somewhat confused. A stuporous patient arouses from sleep after exposure to painful stimuli, exhibits slow verbal responses, and easily lapses into an unresponsive state. A comatose patient remains unarousable with eyes closed. There is no evident response to inner need or external stimuli.

Question:

Aphasia refers to:

 

the inability to produce or understand language.  Correctthe loss of voice.an impairment in volume of the voice.a defect in the muscular control of the speech apparatus.

 

Explanation:

Aphasia refers to a disorder in producing or understanding language. Aphonia refers to a loss of voice that accompanies disease affecting the larynx or its nerve supply. Dysphonia refers to less severe impairment in the volume, quality, or pitch of the voice. Dysarthria refers to a defect in the muscular control of the speech apparatus (lips, tongue, palate, or pharynx).

Question:

A patient is unable to identify the smell of an orange. This inability could reflect an abnormality in cranial nerves:

 

  1. CorrectII.III.VIII.

 

Explanation:

Cranial Nerve I is the olfactory nerve responsible for the sense of smell. To test the sense of smell, the examiner presents the patient with familiar and nonirritating odors. A person should normally perceive odor on each side and correctly identify the source. Cranial Nerves II and III assess vision and pupillary reaction. Cranial Nerve VIII tests the hearing and balance.

Question:

When assessing the cranial nerves, the nurse practitioner instructs the patient to stick out his tongue and move it from side to side. This maneuver would be used to assess which cranial nerve?

 

Cranial Nerve V (CN V)

Cranial Nerve VII (CN VII)Cranial Nerve IX (CN IX)Cranial Nerve XII (CN XII)  Correct

 

Explanation:

Instructing the patient to stick out his tongue and move it from side to side would be used to assess cranial nerve XII (CN XII)-Hypoglossal nerve.

Question:

The level of consciousness that refers to the ability of the patient to respond fully and appropriately to stimuli is known as:

 

obtundation.alertness.  Correctlethargy.stupor.

 

Explanation:

The level of consciousness that refers to the ability of the patient to respond fully and appropriately to stimuli is known as alertness. An obtunded patient opens his eyes, looks at the person speaking to him but responds slowly and appears confused. Lethargy refers to the patient that appears drowsy but can open his eyes, respond to questions, then fall back to sleep. A stuporous patient arouses from sleep only after painful stimuli.

Question:

A sudden brief lapse of consciousness with momentary blinking, staring, or movements of the lips and hands but no falling is consistent with:

 

a myoclonic seizure.an absent seizure.  Correcta myoclonic atonic seizure.a focal seizure with impairment of consciousness.

 

Explanation:

A sudden brief lapse of consciousness, with momentary blinking, staring, or movements of the lips and hands but no falling is consistent with an absent seizure. A patient experiencing a myoclonic seizure manifests sudden, brief, rapid jerks, involving the trunk or limbs. During a myoclonic atonic seizure, the patient experiences a sudden loss of consciousness with falling but no movements. Injury may occur. Focal seizures with impairment of consciousness the person appears confused. Automatisms include automatic motor behaviors such as chewing, smacking the lips, walking about, and unbuttoning clothes.

Question:

Symptoms of a subdural hematoma include:

 

noticeable bleeding between the dura and the skull on X-ray.   Incorrectappropriate responses to questions.noticeable bleeding between the dura and the cerebrum on X-ray.  Correctabsent retinal hemorrhages. APEA NURSING NR 509.

 

Explanation:

Classic symptoms of subdural hemorrhages include: bleeding between the dura and the cerebrum, retinal hemorrhages, confusion, drowsiness, headaches, and possible seizures. Because this is a slow bleed, symptoms may develop slowly over several days or weeks. Bleeding between the dura and the skull are consistent with epidural bleeds.

Question:

Analgesia refers to:

 

decreased sensitivity to pain.increased sensitivity to pain.  Incorrectabsence of pain sensation.  Correctabsence of touch sensation.

 

Explanation:

Analgesia refers to absence of pain sensation; hypalgesia to decreased sensitivity to pain; hyperalgesia to increased sensitivity; and anesthesia to absence of touch sensation.

Question:

While assessing the trigeminal nerve V (CN V) for sensory function, the patient reports a pain sensation on the right cheek. This finding could be consistent with a:

 

bilateral hemispheric disease. central nervous system lesions.  Incorrectcranial nerve disorder.  Correctbrainstem lesion.

 

Explanation:

While assessing the trigeminal nerve, cranial nerve V (CN V) for sensory function, the patient reports a pain sensation on the right cheek. This finding could be consistent with a cranial nerve disorder, such as trigeminal neuralgia.

Question:

When trying to determine the level of consciousness in a patient whose level of consciousness is altered, a comatose patient:

 

opens the eyes and looks at the examiner, responds slowly, and is somewhat confused.appears drowsy but opens the eyes, looks at the examiners, answers the questions, and then falls asleep.arouses from sleep after exposure to painful stimuli, exhibits slow verbal response, and easily lapses into an unresponsive state.remains unarousable with eyes closed.  Correct

 

Explanation:

A comatose patient remains unarousable with eyes closed. There is no evident response to inner need or external stimuli. A lethargic patient appears drowsy but opens the eyes, looks at the examiners, answers the questions, and then falls asleep. An obtunded patient opens the eyes and looks at the examiner, but responds slowly and is somewhat confused. A stuporous patient arouses from sleep after exposure to painful stimuli, verbal responses are slow, and lapses into an unresponsive state.

Question:

Disorders of speech fall into three groups that affect all of the following except the:

 

voice.articulation of speech.written language.  Correctcomprehension of language.

 

Explanation:

Disorders of speech fall into three groups affecting: (1) the voice, (2) the articulation of words, and (3) the production and comprehension of language. The written language is not included in the disorders of speech.

Question:

What is an example of a disease or condition that appears in a dermatomal pattern?

 

Fibromyalgia Shingles  CorrectDiabetic neuropathyReferred pain

 

Explanation:

A band of skin innervated by the sensory root of a single spinal nerve is a dermatome. Symptoms that follow a dermatome (e.g. like pain or a rash) may indicate pathology that involves the related nerve root. Viruses that lie dormant in nerve ganglia (e.g. varicella zoster virus, which causes both chickenpox and herpes zoster shingles), often cause either pain, rash or both in a pattern defined by a dermatome. However, the symptoms may not appear across the entire dermatome. Referred pain usually involves a specific “referred” location so is not associated with a dermatome. Diabetic neuropathy results from nerve damage related to high levels of glucose in the body and the resulting pain or absence of pain does not follow a dermatomal pattern. Fibromyalgia appears to result from neuro-chemical imbalances including activation of inflammatory pathways in the brain which results in abnormalities in pain processing.

Question:

A 80 year old male visits the nurse practitioner for an annual well exam. History reveals two falls in the prior 12 months and difficulty with balance. The next step the nurse practitioner should take is:

 

reassess the patient in 6 months.obtain cognitive and functional assessment.  Correctassess respiratory assessment.assess cardiac function.

 

Explanation:

High-risk older adults, namely those with a single fall in the past 12 months with abnormal gait and balance and those with two or more falls in the prior 12 months, an acute fall, and/or difficulties with gait and balance, require further assessment to determine the reasons for the falls. Obtaining relevant medical history, physical exam, cognitive and functional assessment and determining multifactorial fall risks are essential to the preventing future falls.

Question:

When conducting a neurologic exam, which one of the following assessments is not considered part of the mental status assessment?

 

Level of alertnessCranial Nerve II (CNII)  CorrectAppropriateness of responsesOrientation to time

 

Explanation:

When conducting a neurologic exam, mental status assessment should include evaluation of the level of alertness, appropriateness of responses, and orientation to person, place, and time. Assessing cranial nerve II would be part of the cranial nerve assessment.

Question:

Ataxia, diplopia, and dysarthria can be symptoms associated with all of the following conditions except:

 

posterior fossa tumor.vertebrobasilar transient ischemic attack.polyneuropathy.  Correcthemicranial migraine.  Incorrect

 

Explanation:

Ataxia, diplopia, and dysarthria are suspicious for vertebrobasilar transient ischemic attack or stroke, posterior fossa tumor and vertebrobasilar or hemicranial migraine. Polyneuropathy usually presents as bilateral distal weakness.

Question:

An ischemic stroke is:

 

a transient episode of neurologic dysfunction by focal brain, spinal cord, or retinal ischemia, without acute infarction.   Incorrectan infarction of the central nervous system tissue that may be silent or symptomatic.  Correctthe abrupt onset of motor or sensory deficits.focal or asymmetric weaknesses caused by central and peripheral nerve damage.

 

Explanation:

Ischemic stroke is “an infarction of central nervous system tissue” that may be symptomatic or silent. TIA is now defined as “a transient episode of neurological dysfunction caused by focal brain, spinal cord, or retinal ischemia, without acute infarction.” The other terms are not related to the new definitions.

Question:

During this type of seizure activity, the patient experiences partial seizures that resemble tonic-clonic seizures. The patient may recall the aura and a unilateral neurologic deficit is present during the postictal period. This type of seizure activity is referred to as a:

 

focal seizure with impairment of consciousness.Jacksonian seizure.  Incorrectfocal seizure that becomes generalized.   Correctgrand mal seizure.

 

Explanation:

Focal seizures that become generalized are partial seizures that resemble tonic-clonic seizures. The patient may recall the aura and a unilateral neurologic deficit is present during the postictal period. Focal seizures with impairment of consciousness the person appears confused. Automatisms include automatic motor behaviors such as chewing, smacking the lips, walking about, and unbuttoning clothes. Tonic and then clonic movements that start unilaterally in the hand, foot, or face and spread to other body parts on the same side with the patient remaining conscious are known as Jacksonian seizures. During a grand mal seizure the person loses consciousness suddenly, sometimes with a cry, and the body stiffens into tonic extensor rigidity. Breathing stops, and the person becomes cyanotic. A clonic phase of rhythmic muscular contraction follows.

Question:

When assessing abdominal cutaneous reflexes, the nurse practitioner strokes the lower abdomen, the localized twitch is absent. This finding could be suggestive of a pathologic lesion in which segmented level of the spine?

 

Thoracic 8, 9, and 10Thoracic 10, 11, and 12  CorrectLumbar 5 and Sacral 1Sacral 2, 3, and 4

 

Explanation:

Superficial (cutaneous) reflexes and their corresponding spinal segments include the following: Abdominal reflexes: upper thoracic 8, 9, 10 and lower thoracic 10, 11, 12; Plantar: lumbar 5 and sacral 1; and Anal: sacral 2, 3, 4.

Question:

An example of symmetric weakness is:

 

the right shoulder. the right hand.both arms.  Correctone the right side of the face.

 

Explanation:

There are 4 different patterns of weakness: Proximal, distal, symmetric, and asymmetric. An example of proximal weakness is weakness in the shoulder or hip girdle. Distal weakness occurs in the hands or feet. Symmetric weakness occurs in the same areas on both sides of the body. An asymmetric weakness occurs in a portion of the face or extremity – a form of focal weakness.

Question:

Assessment of a 70-year-old’s ability to maintain personal safety would be most adversely affected by declining function in the:

 

cardiovascular system.respiratory system.sensory perception system.  Correctgastrointestinal system.

 

Explanation:

The sensory system or sensory perception involves vision, touch, taste, smell, and hearing. With the aging process these perceptions are altered and these alterations put the elderly at risk for falls, burns, inability to smell smoke, and the inability to move fast enough to get out of harm’s way. These impact personal safety. Changes in the cardiovascular, gastrointestinal, and respiratory systems do not usually lead to safety issues.

Question:

When assessing an elderly patient for delirium, the recommended instrument with the best supportive data is the:

 

Delirium Rating Scale (DRS).Confusion Assessment Method (CAM).  CorrectMini Mental State Examination (MMSE). Delirium Superimposed on Dementia Algorithm (DSDA).  Incorrect

 

Explanation:

Confusion Assessment Method (CAM), a screening tool specifically designed to detect delirium, is the most recommended because it provides the best supportive data for bedside use and takes 5 minutes to complete. The Delirium Rating Scale (DRS) covers a range of symptoms relating to delirium, and is useful for diagnostic purposes and for assessing severity and distinguishing delirium from other disorders. It is very complex and requires advanced training to administer. The MMSE is the most widely used instrument to test cognitive function. The Delirium Superimposed on Dementia Algorithm recommends a process to assess for delirium for people with a pre-existing dementia.

NR 509 Week 1 Shadow Health History Assignment Completed

Pre-brief

Obtaining an accurate history is the critical first step in determining the etiology of a patient’s problem. A large percentage of the time, you will actually be able to make a diagnosis based on the history alone. The value of the history, of course, will depend on your ability to elicit relevant information. Your sense of what constitutes important data will grow exponentially as you practice your interviewing skills and through increased exposure to patients and illness…………………… Interviewing patients is an art and should remain an essential skill for successful practice.

In this activity, you will interview Tina Jones to collect data to assess Ms. Jones’ condition. You will also have the opportunity to educate and empathize with Tina to engage in effective therapeutic communication; create a problem listusing evidence from the data you collected; prioritize the identified problems to differentiate immediate from non-immediate care; plan how to best address the most important concern with further assessment, interventions, and patient education; and compare your documentation to model documentation.

Ms. Jones is a pleasant, 28-year-old obese African American single woman who presents to establish care and with a recent right foot injury. She is the primary source of the history. Ms. Jones offers information freely and without contradiction. Her speech is clear and coherent and she maintains eye contact throughout the interview.

Reason for visit: Patient presents for an initial primary care visit today complaining of an infected foot wound.

  • | Overview
  • | Transcript
  • | Subjective Data Collection
  • | Objective Data Collection
  • | Education & Empathy
  • | Documentation / Electronic Health Record
  • | Information Processing
  • | Lab Pass: Certificate of Completion

 ORDER NOW FOR INSTRUCTIONS-COMPLIANT, ORIGINAL PAPER

NR 509 Week 1 Shadow Health: Conversation Concept Lab

NR 509 Week 1 Assignment Shadow Health Digital Clinical Experience DCE Orientation Spring 2018

NR 509 Week 1 Quiz Questions and Answers

    A patient tells the FNP that he is very nervous, that he is nauseated, and that he “feels hot”. This type of data would be:
2.     The patient’s record, laboratory studies, objective data, and subjective data combine to form the:
3.     The FNP is reviewing information about evidence-based practice. Which statement best reflects evidence-based practice?
4.     A 59-year-old patient tells the FNP that he has ulcerative colitis. He has been having black stools for the last 24 hours. How would the FNP best document his reason for seeking care?
5.     A 29-year-old woman tells the FNP that she has excruciating pain in her back. Which would be an appropriate response by the FNP to the woman statement?
6.     In recording the childhood illnesses of a patient who denies having had any, which note by the FNP would be most accurate?7.      If a female patient tells the FNP that she has had six pregnancies, with four live births at term and two spontaneous abortions. Her four children are still living. How would the FNP record this information?
8.     If a female patient tells the FNP that she has had six pregnancies, with four live births at term and two spontaneous abortions. Her four children are still living. How would the FNP record this information?
9.     Which of these statements represents subjective data the FNP obtained from the patient regarding the patient’s skin?
10.  The FNP is obtaining a history for a 30-year-old male patient and is concerned about health promotion activities. Which of these questions would be appropriate to use to assess health promotion activities for this patient?
11.  Which statement indicates that the FNP understands the pain experienced by an elderly person?
12.  The FNP is performing a vision examination. Which of these charts is most widely used for visual examination?
13.  During a complete health assessment, how would the FNP test the patients hearing?
14.  The FNP has just completed an examination of a patient’s extra-ocular muscles. When documenting the findings, the FNP should document the assessment of which cranial nerves?
15.  A patient’s uvula rises midline when she says “ahh” and she has a positive gag reflex. The FNP has just tested which cranial nerve?
16.  During an examination the FNP notices that a patient is unable to stick out his tongue. Which cranial nerve is involved with successful performance of this action?
17.  A patient is unable to shrug her shoulders against the FNP‘s resistant hands. What cranial nerve is involved with successful shoulder shrugging?
18.  During an examination, the patient has just successfully completed the finger to nose and rapid alternating movements test and is able to run each heel down the opposite shin. The FNP would conclude that the patient’s___ function is intact
19.  A five-year-old child is in the clinic for checkup. The FNP would expect him to:
20.  When the FNP performs the confrontation test the FNP has assessed:
21.  Which of these statements is true regarding the complete physical assessment?
22.  Which of these statements is true regarding recording of data from the history and physical examination?
23.  Which of these is included in assessment of general appearance?
24.  The FNP is performing a review of symptoms. Which of these questions are appropriate as Health promotion questions to ask during this time?
25.  The FNP is incorporating a person’s spiritual values into the health history. Which of these questions illustrates the community portion of the FICA questions?
26.  The FNP is preparing to complete a health assessment on a 16-year-old girl whose parents have brought her to the clinic. Which instruction would be appropriate for the parents before the interview begins?

BUY A PLAGIARISM-FREE PAPER HERE

 NR 509 Week 2 Shadow Health Respiratory Physical Assessment Spring 2018

Pre Brief
Tina had an asthma episode 2 days ago. At that time she used her albuterol inhaler and her symptoms decreased although they did not completely resolve. Since that incident she notes that she has had ten episodes of wheezing and has shortness of breath approximately every four hours. Tina presents with continued shortness of breath and wheezing. Be sure to ask pertinent questions during the interview about related body systems. This case study will provide the opportunity to carefully assess lung sounds during the physical examination. Be sure to appropriately document your findings using correct medical terminology.

Reason for visit: Patient presents complaining of a recent asthma episode that is not fully resolved.

  • | Overview
    | Transcript
    | Subjective Data Collection
    | Objective Data Collection
    | Education & Empathy
    | Documentation / Electronic Health Record
    | Student Pre-Survey
    | Lifespan Activity
    | Review Questions
    | Self-Reflection Activity

 NR 509 Week 2 Alternate Writing Assignment: Respiratory Summer 2018

Purpose

As a family nurse practitioner, you must possess excellent physical assessment skills. This alternative writing assignment mirrors the discussion content of the debriefing session and will allow the student to expand their knowledge of physical health assessment principles specific to the advanced practice role.

Course Outcomes

This assignment is guided by the following Course Outcomes (COs):

  1. Apply advanced practice nursing knowledge to collecting health history information and physical examination findings for various patient populations. (PO 1, 2)
  2. Differentiate normal and abnormal health history and physical examination findings. (PO 1, 2)
  3. Adapt health history and physical examination skills to the developmental, gender-related, age-specific, and special population needs of the individual patient. (PO 1, 2)

The purposes of this assignment are to: (a) identify and articulate advanced assessment health history and physical examination techniques which are relevant to a focused body system (CO 1), (b) differentiate normal and abnormal findings with regard to a disease or condition that impacts the body system (CO 2), and (c) adapt advanced assessment skills if necessary to suit the needs of specific patient populations (CO 4).

NOTEYou are to complete this alternative writing assignment ONLY if you had not participated or do not plan to participate in a debriefing session for the given week.

Due Date: This alternative written assignment is due no later than the Sunday of the week in which you did not attend the weekly debriefing session. The standard MSN Participation Late Assignment policy applies to this assignment.

Preparing the Paper:

  1. Select a focused body system from the weekly lesson which corresponds with the week of the written assignment.
  2. Carefully read and review the selected body system in your course textbooks.
  3. Incorporate at least onescholarly peer-reviewed journal article that relates to the body system. It may be useful to identify an article that relates to a disease that impacts the body system.
  4. The paper must clearly articulate the relevance of advanced physical assessment skills, techniques, application of advanced practice knowledge, and assessment modification (when necessary) to accommodate for specific patient populations.
  5. Provide concluding statements that should summarize key points of the overall assignment content.
  6. In-text citations and reference page(s) must be written using proper APA format (6thedition).

NR 509 Week 2 Shadow Health HEENT Physical Assessment Assignment Summer 2018

Pre-Brief
For the last week, Tina has experienced sore, itchy throat, itchy eyes, and runny nose. She states that these symptoms started spontaneously and have been constant in nature. … has treated her throat pain with occasional throat lozenges which has “helped a little”. She states that her nose “runs all day” and has clear discharge. … denies cough and recent illness. … denies fevers, chills, and night sweats. This case study will allow you to use standard office equipment to physically examine the patient’s eyes, ears, nose, and throat. You will need to document what you find in the Electronic Health Record (EHR). Be certain to use medically appropriate terminology, such as “erythematous” to describe redness of the skin, mucous membranes, or conjunctiva.

  • | Overview
    | Subjective Data Collection
    | Objective Data Collection
    | Education & Empathy
    | Documentation / Electronic Health Record
    | Lifespan
    | Review Questions
    | Self-Reflection

NR 509 Week 2 Quiz

  1. A mother brings her two month old daughter in for an examination says “my daughter rolled over against the wall and now I have noticed that she has the spot soft on the top of her head, is there something terribly wrong?” The FNP‘s best response would be:
  2. During percussion the FNP knows that a dull percussion note elicited over a lung lobe. This most likely results from:
  3. The patient is unable to differentiate between sharp and dull stimulation to both sides of her face. The FNP suspects Damage to:
  4. When examining the face, the FNP is aware that the two pairs of salivary gland‘s that are accessible to examination are the _____ glands
  5. A patient comes to the clinic complaining of neck and shoulder pain and is unable to turn her head. The FNP suspects damage to cranial nerve ____ and proceeds with the examination by____
  6. When examining a patient’s cranial nerve function, the FNP remembers that the muscles in the neck that are innervated by CN XI are the:
  7. The patient’s laboratory data reveal an elevated thyroxine level. The FNP would proceed with an examination of the _____ gland
  8. A patient says that she has recently noticed a lump in the front of her neck below her “Adams apple” that seems to be getting bigger. During the assessment, the finding that leaves the FNP to suspect that this may not be a cancerous thyroid nodule is that the lump:
  9. The FNP notices that the patient’s submental lymph nodes are enlarged. In an effort to identify the cause of the node enlargement, the FNP would assess the patient’s:
  10. The FNP is aware that the four areas in the body were lymph nodes accessible are the:
  11. A 52-year-old patient describes the presence of occasional floaters or spots moving in front of his eyes. The FNP should know that floaters are usually not significant and are caused by:
  12. The FNP is preparing to assess the visual acuity of a 16-year-old patient. How should the FNP proceed?
  13. A patient’s vision is recorded as 20/30 when the Snellen eye chart is used. The FNP interprets these results to indicate that:
  14. A patient is unable to read even the largest letters on the Snellen chart. The FNP should take which action next:
  15. A patient’s vision is reported as 20/80 in each eye. The FNP interprets this finding to mean that
  16. When performing the corneal light reflex assessment, the FNP notes that the light is reflected at 2 o’clock in each eye. The FNP should
  17. The FNP is performing the diagnostic positions test. Normal findings would be which of these results?
  18. During an assessment of the sclera of an African-American patient, the FNP would consider which of these an expected finding?
  19. A 60-year-old man is at the clinic for an examination. The FNP suspects that he has ptosis of one eye. How should the FNP check for this?
  20. The FNP is doing an assessment on a 21-year-old patient and notices that his nasal mucosa appears pale gray and swollen. What would be the most appropriate question to ask the patient?
  21. The FNP is palpating the sinus areas. If the findings are normal, then the patient should report which sensation?
  22. During an oral assessment of a 30-year-old African-American patient, the FNP notices bluish lips and a dark line along the gingival margin. What would the FNP do in response to these findings
  23. During an assessment of a 20-year-old patient with a three day history of nausea and vomiting the FNP notices dry mucous and deep vertical fissures on the tongue. These findings are reflective of:
  24. The FNP is reviewing the technique of palpating for tactile fremitus with a new graduate. Which statement by the graduate FNP reflects a correct understanding of tactile fremitus?
  25. The FNP student is reviewing physical assessment findings of the HEENT system associated with pregnancy. Which statement by the graduate FNP reflects a correct understanding of expected HEENT changes associated with pregnancy? During pregnancy:

NR 509 Week 2 Quiz Review – (Jarvis 8,9,13,14,15,16,18), (Swartz 4,6,7,8,9,10)

1.       What does dullness when percussing lung fields: Jarvis pg 427

2.       Facial sensation controlled by which CN: Jarvis 283,

3.       Know what two salivary glands are accessible during exam

4.       What CN is being … when pt shrugs shoulders Jarvis 646

5.       What muscles are being …. when …. CN 11 (spinal accessory nerve)

6.       Concern for malignant nodules versus benign lymph nodule

7.       Know what you’d do next if you palpated a submental lymph node: Jarvis pg 253

8.       Define visual acuity

9.       Know what to do if your patient can’t read the largest number on the Snellen chart: Jarvis 289

10.   Example of good visual acuity : Jarvis 289

11.   Example of poor visual acuity: Jarvis 289

12.   What is …. with corneal light reflex-

13.   Know normal variances of sclera : Jarvis 283

14.   Know how to check for Ptosis: Jarvis 292

15.   What does ptosis indicate: Jarvis 292

16.   Nasal fissure of pt with chronic allergies : Jarvis 271

·         Acute allergies : Jarvis p 363 .

17.   What is an abnormal palpation of sinuses: Jarvis 362

·         Normal palpation of sinuses

18.   Know normal variations in gingival margin

19.   Know what a dehydrated oral cavity will look like: Jarvis 387

20.   What is tactile fremitus, how do you test for it and what does it indicate. Jarvis 425

NR 509 Week 3 Alternate Writing Assignment Neurological System Summer 2018

Purpose

As a family nurse practitioner, you must possess excellent physical assessment skills. This alternative writing assignment mirrors the discussion content of the debriefing session and will allow the student to expand their knowledge of physical health assessment principles specific to the advanced practice role.

The purposes of this assignment are to: (a) identify and articulate advanced assessment health history and physical examination techniques which are relevant to a focused body system (CO 1), (b) differentiate normal and abnormal findings with regard to a disease or condition that impacts the body system (CO 2), and (c) adapt advanced assessment skills if necessary to suit the needs of specific patient populations (CO 4).

NOTEComplete this alternative writing assignment ONLY if you had not participated or do not plan to participate in a debriefing session for the given week.

Due Date: Alternative written assignment is due no later than the Sunday of the week in which you did not attend the weekly debriefing session.

Preparing the Paper:

  1. Select a focused body system from the weekly lesson which corresponds with the week of the written assignment.
  2. Carefully read and review the selected body system in your course textbooks.
  3. Incorporate at least onescholarly peer-reviewed journal article that relates to the body system. It may be useful to identify an article that relates to a disease that impacts the body system.
  4. The paper must clearly articulate the relevance of advanced physical assessment skills, techniques, application of advanced practice knowledge, and assessment modification (when necessary) to accommodate for specific patient populations.
  5. Provide concluding statements that should summarize key points of the overall assignment content.
  6. In-text citations and reference page(s) must be written using proper APA format (6thedition).

 NR 509 Week 3 Shadow Health Assessment Musculoskeletal Spring 2018

Subjective:

HPI: Ms. Jones presents to the clinic complaining of back pain that began 3 days ago after she “tweaked it” while lifting a heavy box while helping a friend move. She states that lifted several boxes before this event without incident and does……………………………… She presents today as the pain has continued and is interfering with her activities of daily living.

Social History: Ms. Jones’ job is mostly supervisory, although she does report that she may have to sit or stand for extended periods of time……………. use of tobacco, alcohol, and illicit drugs. She does not exercise.

ROS: General: Denies changes in weight, fatigue, weakness, fever, chills, and night sweats. •

Musculoskeletal: Denies muscle weakness, pain, joint instability, or swelling. She does state that she has difficulties with range of motion…… lower back has impacted her comfort while sleeping and sitting in class…….. numbness, tingling, radiation, or bowel/bladder dysfunction. She denies previous musculoskeletal injuries or fractures. •

Neurologic: Denies loss of sensation, numbness, tingling, tremors, weakness, paralysis, fainting, blackouts, or seizures.

Objective:

ROS: General: Ms. Jones is a pleasant, obese 28-year-old African American woman in no acute distress. She is alert and oriented. She maintains eye contact throughout interview and examination.

Musculoskeletal: Bilateral upper extremities without muscle atrophy or joint deformity. Bilateral upper extremities with full range of motion of shoulder, elbow, and wrist……………….. upper extremity strength equal and 5/5 in neck, shoulders, elbows, wrists, hands. Bilateral lower extremity strength equal and 5/5 in hip flexors, knees, and ankles.

ASSESSMENT: Low back muscle strain related to lifting

  • | Overview 
    | Transcript
    | Subjective Data Collection 
    | Objective Data Collection 
    | Education & Empathy 
    | Documentation
    | Lifespan
    | Review Questions
    | Self-Reflection

NR 509 Week 3 Shadow Health Neurological Physical Assignment Completed

Pre Brief

Two days after a minor, low-speed car accident in which Tina was a passenger, she noticed daily bilateral headaches along with neck stiffness. She reports that it hurts to move her neck, and she believes her neck might … swollen. She did not lose consciousness in the accident and denies changes in level of consciousness since that time. … that she gets a headache every day that lasts approximately 1-2 hours. … occasionally takes 650 mg of over the counter Tylenol with relief of the pain. This case study will allow you the opportunity to examine the patient’s optic nerve via use of the ophthalmoscope as well as assess her visual acuity. You will need to document your findings using appropriate medical terminology. Be sure to assess for foot neuropathy using the monofilament test. Reason for visit: Patient presents complaining of headache.NR 509 Neurological Results | Transcript

  • Interview Questions (113)
NR 509 Neurological Results | Subjective Data Collection: 18 of 20 (90.0%)
NR-509 Neurological Results | Objective Data Collection: 36.75 of 37 (99.32%)
NR 509 Neurological Results | Education & Empathy: 4 of 4 (100.0%)
  1. Symptoms
  2. Cause of Injury
  3. Medications
  4. Vision

NR 509 Neurological Results | Documentation / Electronic Health Record

NR 509 Neurological Results | Pre-Survey Lifespan

  1. Tina’s three-year-old neighbor presents to the clinic with fever, neck pain, headache, and confusion. He has no symptoms of an upper respiratory infection. The parents mention that they do not believe in immunizations. Based on the information given, what diagnosis is of the greatest concern? What is your next action?
  2. Tina’s 83-year-old great uncle forgets where he is during his yearly check-up. He doesn’t remember if he’s had memory problems before and no family members came to your office with him. List your differential diagnosis. What assessments would you perform?

NR 509 Neurological Results | Review Questions

  1. To assess spinal levels L2, L3 and L4 in Tina, which deep tendon reflexes would have to be tested?
  2. Imagine that you were preparing to irrigate a Foley catheter of a patient with a spinal cord injury at T4 in a urology clinic. Upon moving the leg bag, the patient became suddenly flushed and diaphoretic above the nipple line. What would you suspect was happening?
  3. Which of the following is not a common symptom of Parkinson’s disease?
  4. Name at least three ways to assess cerebellar function during a physical exam.
  5. If Tina had a fever and photophobia, you would have had to test for meningitis. Describe how you would have tested for the Kernig’s sign
  6. Suppose you assessed pain sensation over Tina’s left foot, and noticed that she had decreased sensation. How would you have proceeded with your exam?

NR 509 Neurological Results | Self-Reflection

  1. Explicitly describe the tasks you undertook to complete this exam.
  2. Explain the clinical reasoning behind your decisions and tasks.
  3. Identify how your performance could be improved and how you can apply “lessons learned” within the assignment to your professional practice.

NR 509 Week 4 Quiz Advanced Physical Assessment Chamberlain 2018

1.       A 35-year-old man is seen in the clinic for an infection in his left foot. Which of these findings should the FNP expect to see during an assessment of this patient?

2.       The direction of blood flow through the heart is best described by which of these

3.       In assessing the carotid artery’s of an older patient with cardiovascular disease, the FNP would

4.       When listening to heart sounds the FNP knows that the valve closures that can be heard best at the base of the heart are

5.       The sack that surrounds and protects the heart is called the

6.       When assessing a newborn infant who is five minutes old the FNP knows that which of these statements would be true?

7.       The FNP is performing an assessment on an adult. The adults vital signs are normal and capillary refill is five seconds. What should the FNP do next?

8.       During an assessment of an older adult the FNP should expect to notice which finding as normal physiologic change associated with aging process?

9.       The mother of a three month old infant states that her baby has not been gaining weight. With further questioning the FNP finds that the infant falls asleep after nursing and wakes up after a short amount of time hungry again. What other information with the FNP want to have?

10.   In assessing a patient’s major risk factors for heart disease which would the FNP want to include when taking a history?

11.   The FNP is … the pulses of a patient who has been admitted for untreated hyperthyroidism. The FNP should expect to find a____pulse

12.   A patient complains of leg pain that wakes him at night. He states that he has been having problems with his legs. ….. his legs when they are … and disappears when he dangles them. He recently noticed a sore on the inner aspect of his right ankle. On the basis of this history information the FNP interprets that the patient is most likely experiencing

13.   During an assessment the FNP uses the profile sign to detect

14.   Which of these statements describes the closure of the valves in a normal cardiac cycle?

15.   When performing a peripheral vascular assessment on a patient the FNP is unable to palpate the ulnar pulses. The patient skin is warm and capillary refill is normal. The FNP should next

16.   A 67-year-old patient states that he “recently began have pain in his left calf when climbing the 10 stairs to his apartment”. This pain is relieved by sitting for about two minutes then he’s able to resume activities. The FNP interprets this patient is most likely experiencing

17.   In assessing a 70-year-old man the FNP finds the following blood pressure 140/100 mmHg, heart rate 104 and slightly irregular, split S2. Which of these findings can… by expected hemodynamic changes related to age?

18.   The FNP is examining the lymphatic system of a healthy three year old child. Which finding should the FNP expect?

19.   The FNP is preparing to perform modified Allen test. Which is an appropriate reason for this test?

20.   A 25-year-old woman is in her fifth month of pregnancy has a blood pressure of 100/70 mmHg. In reviewing her previous exam the FNP notes that her blood pressure in her second month was 124/80 mmHg. When evaluating this change what does the FNP know to be true?

21.   Findings from an … of a 70-year-old patient with swelling in his ankles include jugular venous pusations, 5 cm above the sternal angle when the head of his bed is …. 45°. The FNP knows that this finding indicate:

22.   The component of the conduction system referred to as the pacemaker of the heart is the

23.   The FNP is reviewing anatomy and physiology of the heart. Which statement best … by atrial kick?

24.   A 45-year-old man is in the clinic for a routine physical. During history the patient states he has been having difficulty sleeping. I’ll be sleeping great and then I wake up and feel like I can’t catch my breath. The FNP‘s best response to this would be

25.   When assessing a patient the FNP notes that the left femoral pulse as diminished 1+/4+. What should the FNP do next?

NR 509 Week 4 Shadow Health Cardiovascular Physical Assessment Assignment Summer 2018

Overview

Transcript

| Subjective Data Collection

| Objective Data Collection

| Education & Empathy

| Documentation

| Lifespan

| Review Questions

| Self-Reflection

NR 509 Week 4 Shadow Health Chest Pain Physical Assessment Assignment Summer 2018

Overview

Transcript

| Subjective Data Collection

| Objective Data Collection

| Education & Empathy

| Documentation

Document: Provider Notes

Document: Vitals

| Self-Reflection

NR 509 Week 4 Quiz Advanced Physical Assessment Chamberlain 2018

1.       A 35-year-old man is seen in the clinic for an infection in his left foot. Which of these findings should the FNP expect to see during an assessment of this patient?

2.       The direction of blood flow through the heart is best described by which of these

3.       In assessing the carotid artery’s of an older patient with cardiovascular disease, the FNP would

4.       When listening to heart sounds the FNP knows that the valve closures that can be heard best at the base of the heart are

5.       The sack that surrounds and protects the heart is called the

6.       When assessing a newborn infant who is five minutes old the FNP knows that which of these statements would be true?

7.       The FNP is performing an assessment on an adult. The adults vital signs are normal and capillary refill is five seconds. What should the FNP do next?

8.       During an assessment of an older adult the FNP should expect to notice which finding as normal physiologic change associated with aging process?

9.       The mother of a three month old infant states that her baby has not been gaining weight. With further questioning the FNP finds that the infant falls asleep after nursing and wakes up after a short amount of time hungry again. What other information with the FNP want to have?

10.   In assessing a patient’s major risk factors for heart disease which would the FNP want to include when taking a history?

11.   The FNP is … the pulses of a patient who has been admitted for untreated hyperthyroidism. The FNP should expect to find a____pulse

12.   A patient complains of leg pain that wakes him at night. He states that he has been having problems with his legs. ….. his legs when they are … and disappears when he dangles them. He recently noticed a sore on the inner aspect of his right ankle. On the basis of this history information the FNP interprets that the patient is most likely experiencing

13.   During an assessment the FNP uses the profile sign to detect

14.   Which of these statements describes the closure of the valves in a normal cardiac cycle?

15.   When performing a peripheral vascular assessment on a patient the FNP is unable to palpate the ulnar pulses. The patient skin is warm and capillary refill is normal. The FNP should next

16.   A 67-year-old patient states that he “recently began have pain in his left calf when climbing the 10 stairs to his apartment”. This pain is relieved by sitting for about two minutes then he’s able to resume activities. The FNP interprets this patient is most likely experiencing

17.   In assessing a 70-year-old man the FNP finds the following blood pressure 140/100 mmHg, heart rate 104 and slightly irregular, split S2. Which of these findings can… by expected hemodynamic changes related to age?

18.   The FNP is examining the lymphatic system of a healthy three year old child. Which finding should the FNP expect?

19.   The FNP is preparing to perform modified Allen test. Which is an appropriate reason for this test?

20.   A 25-year-old woman is in her fifth month of pregnancy has a blood pressure of 100/70 mmHg. In reviewing her previous exam the FNP notes that her blood pressure in her second month was 124/80 mmHg. When evaluating this change what does the FNP know to be true?

21.   Findings from an … of a 70-year-old patient with swelling in his ankles include jugular venous pusations, 5 cm above the sternal angle when the head of his bed is …. 45°. The FNP knows that this finding indicate:

22.   The component of the conduction system referred to as the pacemaker of the heart is the

23.   The FNP is reviewing anatomy and physiology of the heart. Which statement best … by atrial kick?

24.   A 45-year-old man is in the clinic for a routine physical. During history the patient states he has been having difficulty sleeping. I’ll be sleeping great and then I wake up and feel like I can’t catch my breath. The FNP‘s best response to this would be

25.   When assessing a patient the FNP notes that the left femoral pulse as diminished 1+/4+. What should the FNP do next?

NR 509 Week 5 Shadow Health Abdominal Pain Physical Assessment Assignment Summer 2018

·  Overview

·  | Transcript

·  | Subjective Data Collection

·  | Objective Data Collection

·  | Education & Empathy

·  | Documentation

·         Document: Provider Notes

·         Document: Vitals

·  | Self-Reflection

NR 509 Week 5 Shadow Health Gastrointestinal Physical Assessment Assignment Summer 2018

Transcript

Subjective Data Collection

| Objective Data Collection

| Education & Empathy

| Documentation 

| Lifespan 

| Review Questions 

| Self-Reflection

NR 509 Week 5 Quiz 2 Practice Versions Advanced Physical Assessment Chamberlain 2018

1.       An older patient has been diagnosed with pernicious anemia. The FNP knows that this condition could be related to

2.       ….. examining a patient who tells the FNP “I sure sweat a lot especially on my face and feet but it doesn’t have an odor”. The FNP knows that this could … related to

3.       During an abdominal assessment the FNP elicits tenderness on light palpation in the right lower quadrant. The FNP interprets that this finding could indicate a disorder which of these structures?

4.       An Inuit visiting Nevada from anchorage has come to the clinic in July during the hottest part of the day. It so happens that the clinics air conditioning is broken and the temperature is very hot. The FNP knows that which of these statements is true about the Inuit sweating tendencies?

5.       The FNP notices that a patient has black, tarry stool and recalls that a possible cause would …

6.       ….. an abdominal …. deep palpation is used to determine

7.       The FNP is assessing the abdomen of an aging adult. Which of these statements regarding an aging adult and abdominal assessment is true?

8.       During examination the FNP finds that a patient has excessive dryness of the skin. The best term to describe this condition is

9.       A FNP notices that a patient has ascites, which indicates the presence of

10.   The FNP is performing percussion during an abdominal assessment. Percussion notes during the abdominal assessment may include

11.   The FNP is caring for a black child who has … with marasmus. The FNP would expect to find the

12.   … patient’s medical record that the patient has a lesion that is confluent in nature. On examination the FNP would expect to find

13.   The FNP is assessing the abdomen of a pregnant woman who is complaining of having acid indigestion all the time. The FNP knows that esophageal reflux during pregnancy can cause

14.   The patient has abdominal borborygmi. The FNP knows that this term refers to

15.   A patient has a terrible itch for several months that he … scratching continuously. On examination the FNP might expect to find

16.   During aging process, the hair can look gray or white and begin to feel thin and fine. The FNP knows that this occurs because of a decrease in number of functioning

17.   The FNP notices that a patient has a solid, elevated, circumscribed lesion that is less than 1 cm in diameter. When documenting this finding the FNP would report this as a

18.   During an abdominal assessment the FNP would consider which of these findings as normal?

19.   A 52 -year-old Woman has a papule on her nose that has a rounded pearly border and a central red ulcer. She said she first noticed it several months ago and that it is slowly growing larger. The FNP suspects which condition?

20.   The FNP is listening to bowel sounds. Which of these statements is true about bowel sounds?

21.   The FNP is watching a new graduate FNP perform auscultation of a patient abdomen. Which statement by the new graduate shows a correct understanding of the reason auscultation proceeds percussion and palpitation of the abdomen?

22.   A patient is complaining of a sharp pain along the costovertebral angles. The FNP knows that this symptom is most often indicative of

23.   The patient is … of having inflammation of the gallbladder or cholecystitis. The FNP should conduct which of these techniques to assess for this condition?

24.   A newborn infant is in the clinic for a well baby check. The FNP observes the infant for possibility of fluid loss because of which these factors?

25.   During an assessment of a newborn infant, the FNP recalls that pyloric stenosis would … manifested by

NR 509 Week 5 Quiz: Skin, Hair, Abdomen

1.       Question: The FNP is assessing the abdomen of an aging adult. Which of these statements regarding the agind adult and abdominal assessment is true?

2.       Question: The FNP notices that a patient has had a black tarry stool and recalls that a possible cause would be

3.       Question: The FNP knows that during an abdominal assessment deep palpitation is used to determine

4.       Question: A patient has abdominal borboygmi. The FNP knows that this term refers to

5.       Question: During an abdominal assessment, the FNP would consider which of these findings as normal?

6.       Question: The FNP is caring for a black child who has been diagnosed with marasmus. The FNP would expect to find the

7.       Question: An Inuit visiting Nevada from Anchorage has come to the clinic in July during the hottest part of the day. It so happens that the clinic’s air conditioning is broken and the temperature is very hot. The FNP knows that which of these statements is true about the Inuit sweating tendencies?

8.       Question: The FNP is performing percussion during an abdominal assessment. Percussion notes heard during the abdominal assessment may include:

9.       Question: During an abdominal assessment, the FNP elicits tenderness on light palpation in the right lower quadrant. The FNP interprets that this finding could indicate a disorder of which of these structures?

10.   Question: A patient has a terrible itch for several months that he has been scratching continuously. On examination, the FNP might expect to find

11.   Question: The FNP is listening to bowel sounds. Which of these statements is true of bowel sounds?

12.   Question: A patient is complaining of a sharp pain along the costovertebral angles. The FNP knows that this symptom is most often indicative of

13.   Question: The FNP notices that a patient has a solid, elevated, circumscribed lesion that is less than 1 cm in diameter. When documenting this finding, the FNP would report this as a

14.   Question: A patient is suspected of having inflammation of the gallbladder, or cholecystitis. The FNP should conduct which of these techniques to assess for this condition?

15.   Question: The FNP just noted from a patient’s medical record that the patient has a lesion that is confluent in nature. On examination, the FNP would expect to find

16.   Question: During an examination, the FNP finds that a patient has excessive dryness of skin. The best term to describe this condition is

17.   Question: An older patient has been diagnosed with pernicious anemia. The FNP knows that this condition could be related to

18.   Question: A 52 year old woman has a papule on her nose that has rounded, pearly borders and a central red ulcer. She said she first noticed it several months ago and that it has slowly grown larger. The FNP suspects which condition?

19.   Question: The FNP is watching a new graduate FNP perform auscultation of a patient’s abdomen. Which statement by the new graduate shows a correct understanding of the reason auscultation precedes percussion and palpation of the abdomen?

20.   Question: A newborn infant is in the clinic for a well-baby check. The FNP observes the infant for the possibility of fluid loss because of which of these factors?

NR 509 Week 5 Quiz Review

1.       Tenderness in RLQ is concerning for

2.       Inuit people tend to sweat more on face than trunk and extremities when exposed to heat

3.       spot on face with round pearly boarder with central red lesion concerning for

4.       Murphy sign= positive with deep palpation causes pain on inspiration

5.       Why do you auscultate first in abd assessment

6.       Tympany in the umbilical area

7.       Pt with decreased gastric secretions is at increase

8.       Another name for hyperactive bowels

9.       Black tarry stool concerning

10.   Lichenification caused by

11.   A papule is

12.   Older adults have

13.   CVA tenderness

14.   Confluent lesions

15.   Dryness of the skin

16.   Newborns are at greater risk of fluid loss

17.   High pitched irregular bowel sounds

18.   Changes in hair texture and color of an AA child with Marasmus

19.   Know how to assess for enlarged organs in the abd region

20.   Know three sounds heard in percussion

NR 509 Week 6 Shadow Health Mental Health Physical Assessment Assignment Summer 2018

NR 509 Mental Health Results | Transcript

NR 509 Mental Health Results | Subjective Data Collection

NR 509 Mental Health Results | Education and Empathy

NR 509 Mental Health Results | Documentation

NR 509 Mental Health Results | Life Span

NR 509 Mental Health Results | Review Questions

NR 509 Mental Health Results | Self Reflection

NR 509 Week 7 Shadow Health Comprehensive Health History and Physical Assessment Assignment

NR 509 Comprehensive Assessment Results: Experience Overview

  1. Digital Clinical Experience Score: Score 98.4%
  2. Student Performance Index: 121 out of 123

NR 509: Comprehensive Assessment Results: Transcript (FNP Student Conducting Interview)

NR-509 Comprehensive Assessment Results: Subjective Data Collection: 50 of 50 (100%)

NR 509: Comprehensive Assessment Results: Objective Data Collection: 71.0 of 73 (97.3%)

NR-509 Comprehensive Assessment Results: Documentation / Electronic Health Record

NR 509 Comprehensive Assessment Results: Plan My Exam

Your Final Plan

  1. Head and neck,
  2. Anterior chest,
  3. Posterior chest,
  4. Abdomen,
  5. Upper extremities,
  6. Lower extremities,
  7. Full body

NR 509: Comprehensive Assessment Results: Self-Reflection

  1. Explicitly describe the tasks you undertook to complete this exam.
  2. Explain the clinical reasoning behind your decisions and tasks.
  3. Identify how your performance could be improved and how you can apply “lessons
  4. learned” within the assignment to your professional practice.

NR 509 Week 7 Assignment Immersion Completion Spring 2018

__________, my name is ____________, I will be doing your exam today. I’ll begin with inspecting your face. I’ll note that I don’t see any discolorations or lesions & the head is midline & symmetrical.

  1. LYMPH NODES: Next, I’ll palpate the lymph nodes. I’ll begin with the preauricular lymph nodes & postauricular lymph nodes. Next the occipital lymph nodes. I’ll move forward to palpate the tonsillar lymph nodes, submandibular & submental. I’m palpating the anterior cervical lymph nodes & posterior cervical lymph nodes & lastly the supraclavicular lymph nodes. I don’t feel any enlargement & they’re equal bilaterally.
  2. FACE : I’m testing trigeminal nerve, which is cranial nerve # 5. Palpating over the masseter muscle as the pt clenches the jaw. I don’t feel any distortions & my pt has great strength. Now, I’m testing the sensory portion of the trigeminal nerve. I’ll ask my pt to close your eyes & let me know where you feel my touch. (>> Forehead, right cheek, left cheek, chin, nose)………. which is cranial nerve # 7. I’m going to ask you to do some facial expressions. I’m going to have you smile, next, frown for me. Now raise your eyebrows & puff up your cheeks, pucker your lips. I notice all expressions have bilateral symmetry.
  3. EARS
  4. EYES
  5. NOSE
  6. MOUTH
  7. NECK & THROAT
  8. HEART/CHEST((pt sitting))
  9. UPPER EXTREMITIES
  10. ABDOMEN((have pt lie down))
  11. LOWER EXTREMITIES

NR 509 Week 6 Quiz Practice 2 Versions Advanced Physical Assessment Chamberlain 2018

1.       A woman has come to the clinic to seek help with a substance-abuse problem. She admits to using cocaine just before arrival. Which of these assessment findings would the FNP expect to find when examining the woman?

2.       A 63-year-old Chinese American man enters the office with complaints of chest pain, shortness of breath, and palpitations. Which statement most accurately reflect the FNPs best course of action?

3.       The FNP is planning to assess new memory with the patient. The best way for the FNP to do this would be

4.       During the health history the FNP asks a female patient “how many alcoholic drinks do you have a week?” Which answer by the patient would indicate at risk drinking?

5.       Symptoms such as pain are often influenced by a person’s cultural heritage. Which of the following is a true statement regarding pain?

6.       The FNP suspect abuse when a 10-year-old child is taken to the urgent care center for leg injury. The best way to document the history and physical findings is to

7.       During a mental status assessment, which question by the FNP would best assess a persons judgment?

8.       The FNP is performing a mental status assessment on a five-year-old girl. Her parents are undergoing a bitter divorce and are worried about the effect it is having on their daughter. Which action or statement might leave FNP to be concerned about the girls mental status?

9.       During mental status examination, the FNP wants to assess a patient’s affect. The FNP should ask the patient which question?

10.   During an examination FNP notices a patterned injury on a patients back. Which of these would cause such an injury?

11.   The FNP is aware that intimate partner violence screening should occur with which situation?

12.   Which statement is best for the FNP to use when preparing to administer the abuse assessment screen?

13.   The FNP is conducting a class on alcohol and the effects of alcohol on the body. How many standard drinks (each containing 12 g alcohol) Per day are associated with increased deaths from cirrhosis, cancers of the mouth, esophagus and injuries in men?

14.   A woman who has just discovered that she is pregnant is in the clinic for her first obstetric visit. She asked the FNP how many drinks a day safe for my baby? The FNP‘s best response is

15.   The FNP is performing the Denver II screen test on a 12 month old infant during a routine well child visit. The FNP should tell the infants parents that the Denver II

16.   Which term refers to a one produced by tearing or splitting of body tissue usually from blunt impact of a bony surface

17.   When reviewing the use of alcohol by older adults the FNP notes that the older adults have several characteristics that can increase the risk of alcohol use which would increase the bioavailability of alcohol in the blood for longer periods of time in the older adult?

18.   The FNP is reviewing concepts of cultural aspects of pain. Which statement is
true regarding pain?

19.   The FNP is planning to assess a child using behavioral checklist. This tool is most appropriate for a(an)

20.   The FNP is assessing orientation in a 79-year-old patient. Which of these responses … leave the FNP to … that the patient is … ?

21.   As a mandatory reporter of elder abuse, which of these must be present before an FNP notifies the authorities?

22.   A 30-year-old female patient is describing feelings of hopelessness and depression. She has attempted self-mutilation and has a history of prior suicide attempts. She describes difficulty sleeping at night and has lost 10 pounds in the past month. Which of these statements or questions is the FNP‘s best response in this situation?

23.   For persons age 12 years and older which of these illicit substances was one of the most commonly used?

24.   Which of these individuals would the FNP consider at highest risk for suicide attempt?

25.   The FNP is …. a patient who has … for cirrhosis of the liver secondary to chronic alcohol use. During the physical … the FNP looks for cardiac problems that are … with chronic heavy use of alcohol such as

NR 509 Week 7 Quiz Practice 2 Versions Advanced Physical Assessment Chamberlain 2018

1.       During an examination of an aging male the FNP recognizes that normal changes to expect would be:

2.       During a health history, a 22-year-old woman asks “can I get that vaccine for HPV? I have gentle warts and I’d like them to go away!” What is the FNP‘s best response?

3.       During a speculum inspection of the vagina the FNP would expect to see what at the end of the vaginal canal?

4.       A 62-year-old man is experiencing fever, chills, malaise, urinary frequency and urgency. He also reports urethral discharge and a dull aching pain in the perineal and rectal area. The symptoms are most consistent with which of the following?

5.       When performing a genital examination on a 25-year-old man the FNP notices deeply pigmented, wrinkled scrotal skin with large sebaceous follicles. On the basis of this information the FNP would:

6.       The mother of a 10-year-old boy asks the FNP to discuss the recognition of puberty. The FNP should reply by saying:

7.       The uterus is usually … tilting forward and superior to the bladder. This position is known as

8.       A male patient with possible fertility problems asks the FNP where sperm is produced. The FNP knows that sperm production occurs in

9.       A 15-year-old boy is seen in the clinic for complaints of dull pain and pulling in the scrotal area. On examination the FNP palpates a soft, irregular mass posterior to and above the testes on the left. This mass collapses when the patient is supine in refills when he is up right. This description is consistent with:

10.   In performing an assessment of a woman’s axillary lymph system the FNP should assess which of these nodes?

11.   A patient contacts the office and tells the FNP that she is worried about her 10-year-old daughter having breast cancer. She describes a unilateral enlargement of the right breast with associated tenderness. She is worried because the left breast is not enlarged. What would be the FNP‘s best response?

12.   An 11-year-old girl is in the clinic for a sports physical. The FNP notices that she has begun to develop breasts, and during the conversation the girl reveals that she is worried about her development. …. which of these techniques to best assist the young girl in understanding the expected sequence for development? The FNP should:

13.   A 54-year-old woman who has just completed menopause is in the clinic today for yearly physical examination. Which of these statements should the FNP include in patient education? A post menopausal woman:

14.   A 62-year-old man states that his doctor told him that he has an inguinal hernia. He asks the FNP to explain what a hernia is……:

15.   When performing a genital assessment on a middle-age man, the FNP notices multiple soft, moist, painless papules in the shape of cauliflower like patches scattered across the shaft of the penis. These lesions are characteristics of:

16.   If a patient reports a recent breast infection, then the FNP should expect to find_____node enlargement

17.   A 9-year-old girl is in the clinic for a sports physical. After some initial shyness she finally asked “am I normal? I don’t seem to need a bra yet, but I have some friends who do. What if I never get breasts?” The FNP‘s best response would be:

18.   Which of these statements about the testes this true?

19.   During an examination FNP observes a female patients vestibule and expect to see the

20.   A 14-year-old girl is anxious about not having reached menarche. When taking history, the FNP should ascertain which of the following? The age:

21.   A woman who is 22 weeks pregnant has a vaginal infection. She tells the FNP that she is afraid that the infection will hurt the fetus. The FNP knows that which of these statements is true?

22.   In performing a breast examination the FNP knows that it is especially important to examine the upper outer quadrant of the breast. The reason for this is that the upper outer quadrant

23.   An accessory glandular structure for the male genital organs is the

24.   Which of these statements is true regarding the penis?

25.   A woman who is 8 weeks pregnant is in the clinic for a check up. The FNP reads on her chart that her cervix is … cyanotic. The FNP knows that the woman is exhibiting____sign and _____sign

NR 509 Week 8 Reflection MSN program outcome 1 and the MSN Essential

Reflect back over the past eight weeks and describe how the achievement of the course outcomes in this course have prepared you to meet the MSN program outcome #1 and the MSN Essential I.

NR 509 Week 1 Discussion (Patient Information: JDS, 30, M, African American, Cigna)

NR.509 Week 2 Discussion (Patient Information: KMW, 27, F, African American, Aetna)

NR 509 Week 3 Discussion (Patient Information: SJS 33, F, African American, Cigna)

Using a friend, family member, or colleague, perform a neurovascular (include all cranial nerves), musculoskeletal, and cardiopulmonary (includes the heart, lungs, and peripheral vasculature) exam. Document the physical examination findings in the SOAP note format.

Even though your patient may have abnormal findings, you must document the expected normal exam findings for the system. If you would like to include the abnormal findings they should be noted in parenthesis next to the normal expected findings. The complete subjective and objective sections must be included………… plan portion of the SOAP note, but these sections will not be graded.

You should devise a chief complaint so that you may document the OLDCART (HPI) data. You must use the chief complaint of headache, back pain, and cough. ______ ROS based on the patient’s chief complaint and the body systems being examined. Refer to the SOAP Note Format document in Course Resources as necessary. This will be the same format that faculty will follow during the immersion weekend.

* There are videos of the exams to be performed at immersion in Modules → Introduction and Resources→ Immersion section. Also the immersion evaluation forms are located in the Course Resources section. They should be reviewed and practiced often.

NR.509 Week 4 Discussion (Patient Information: NJL, 22, M, African American, Cigna)

Using a friend, family member, or colleague, perform a detailed men’s health history. Document the history and the expected normal physical examination findings in the SOAP note format.

Even though your patient may have abnormal findings, you must document the expected normal exam findings for the system. If you would like to include the abnormal findings they should be noted in parenthesis next to the normal expected findings. The complete subjective and objective sections must be included…………..assessment and plan portion of the SOAP note, but these sections will not be graded.

You should devise a chief complaint so that you may document the OLDCART (HPI) data. You must use the chief complaint of penile discharge, rectal bleeding, or frequent urination. You should also focus the ROS based on the patient’s chief complaint and the body systems being examined. Refer to the SOAP Note Format document in Course Resources as necessary. This will be the same format that faculty will follow during the immersion weekend.

* There are videos of the exams to be performed at immersion in Modules → Introduction and Resources→ Immersion section. Also the immersion evaluation forms are located in the Course Resources section. They should be reviewed and practiced often.

NR 509 Week 5 Discussion (Patient Information: JDS, 21, M, African American, Blue Cross Blue Shield)

Using a friend, family member, or colleague, perform a detailed women’s health history. Document the history and the expected normal physical examination findings in the SOAP note format.

Even though your patient may have abnormal findings, you must document the expected normal exam findings for the system. If you would like to include the abnormal findings they should be noted in parenthesis next to the normal expected findings. The complete subjective and objective sections must be included.  You may include the assessment and plan portion of the SOAP note, but these sections will not be graded.

You should devise a chief complaint so that you may document the OLDCART (HPI) data. You must use the chief complaint of burning while urinating, irregular menstrual cycle, or pelvic pain………. focus the ROS based on the patient’s chief complaint and the body systems being examined. Refer to the SOAP Note Format document in Course Resources as necessary. This will be the same format that faculty will follow during the immersion weekend.

* There are videos of the exams to be performed at immersion in Modules → Introduction and Resources→ Immersion section. Also the immersion evaluation forms are located in the Course Resources section. They should be reviewed and practiced often.

NR 509 Focused Exam Cough Assignment Completed Shadow Health

Introduction

Daniel “Danny” Rivera is an 8-year-old boy who comes to the clinic with a cough. Students determine if Danny is in distress, explore the underlying cause of his cough, and look for related symptoms in other body systems.

Case Highlights
  • Ask about a variety of psychosocial factors related to home life, such as second-hand smoke exposure
  • Observe non-verbal cues as Danny presents with intermittent coughing and visible breathing difficulty
  • Rule out asthma, a common childhood affliction, by examining Danny
NR 509 Focused Exam Cough | Transcript
  • Interview Questions (76)
  • Statements (8)
  • Exam Actions (96)

NR 509 Focused Exam Cough | Subjective Data Collection: 20 of 20 (100.0%)

NR-509 Focused Exam Cough | Objective Data Collection: 12.6 of 13 (96.92%)

NR 509 Focused Exam Cough | Education & Empathy:  4 of 5 (80.0%)

  • Symptoms
  • Medications
  • Vitamins
  • Secondhand Smoke
  • Family History

NR 509 Focused Exam-Cough | Documentation / Electronic Health Record

  • Document: Vitals
  • Document: Provider Notes

NR 509 Focused Exam Cough | Self-Reflection

  • Explicitly describe the tasks you undertook to complete this exam
  • Explain the clinical reasoning behind your decisions and tasks
  • Identify how your performance could be improved and how you can apply “lessons learned” within the assignment to your professional nursing practice.
+1 (315) 636-5076
WhatsApp chat +1 (315) 636-5076
www.OnlineNursingPapers.com
We will write your work from scratch and ensure it's plagiarism-free, you just submit the completed work.


WHATSAPP US, WE'LL RESPOND
+1 (315) 636-5076
WhatsApp chat +1 (315) 636-5076
www.OnlineNursingPapers.com
We will write your work from scratch and ensure it's plagiarism-free, you just submit the completed work.


WHATSAPP US, WE'LL RESPOND